HELPPP EDCITEEE !!!!!!!!!!!!!!!!!!!!!

HELPPP EDCITEEE !!!!!!!!!!!!!!!!!!!!!

Answers

Answer 1
105.6yds2

Explanation:
The formula for area of triangle is:
1/2 • base • height
In this case, it is:
1/2 • 17.6 yd • 12 yd
= 105.6 yds2

MARK ME BRAINLIEST TYSM!
Answer 2
yup what the other person said

Related Questions

1;3;5;7…(determine the value of the 19th term of the sequence)​

Answers

Answer:

37

Step-by-step explanation:

fistly find how much different

3-1=2 5-3=2 7-5=2

so for the next and next value just plus two until the 19th then you will get 37

Answer:    37

=============================================================

Explanation:

a = 1 is the first term

d = 2 is the common difference, since we add 2 to each term to get the next term.

The nth term is a+d(n-1) which becomes 1+2(n-1). That simplifies to 2n-1

The last step is to plug in n = 19 and you should get 2(19)-1 = 37

)Which statement best describes the area of the triangle shown below?

A coordinate grid is shown with a triangle.. The base is 4 units, and the height is 4 units.

It is one-half the area of a rectangle of length 4 units and width 2 units.
It is one-half the area of a square of side length 4 units.
It is twice the area of a rectangle of length 4 units and width 2 units.
It is twice the area of a square of side length 4 units.

Answers

9514 1404 393

Answer:

  (b)  It is one-half the area of a square of side length 4 units.

Step-by-step explanation:

The triangle described can be formed by drawing the diagonal through a square 4 units on a side. Such a diagonal cuts the square cleanly in half, so the area of the triangle is half the area of the square.

Answer:

B

Step-by-step explanation:

Hope you have a nice day! :o)

GCF of 9x^2y^2 and 5x^2y^3

Answers

Answer:

x^2y^2.

Step-by-step explanation:

Can somone help me solve this

Answers

Answer:

solid shade above

Step-by-step explanation:

graph line same way you would if it was an equal sign

all the y's that are greater are above the line

its solid because is greater than OR equal to. Equal to includes the line itself as a set of solutions

What is the MEDIAN or Q2 of the data set: 24, 25, 29, 30, 31, 31, 32, 34, 34? ​

Answers

Answer:

31

Step-by-step explanation:

Median = middle number in terms of value

Usually the first step is to list the numbers in order from least to greatest however the numbers are already listed in order from least to greatest

24, 25, 29, 30, 31, 31, 32, 34, 34

We the just go to the middle number

The middle number would be 31

So we can conclude that the median is 31

Will Mark Brainlest Help pls ​

Answers

Answer:

13

HOPE THIS HELPS YOU......

I need help plzzz
How can you construct the perpendicular bisector of segment RS by folding a piece of paper

Answers

9514 1404 393

Answer:

  (c)  fold it so R lies on top of S

Step-by-step explanation:

The crease will be the perpendicular bisector of RS when R and S are equidistant from it. That can be accomplished by folding the paper so points R and S lie on top of each other.

Insurance companies need to maintain a certain amount in reserved funds in order to pay anticipated claims. The average monthly claim amount for the last 60 months for company A was $7,500,000 and the (sample) standard deviation was $1,200,000.
(a) Find a 95% upper confidence bound on the average monthly claim amount.
(b) The regulations on the reserves will be strengthened in the near future. According to the new regulations, insurance companies that do not have sufficient amount in reserve will be subject to a significant penalty. Company A wants to adjust the target reserve amount accordingly, by computing a new upper bound on the average monthly claim amount. Should the company recalculate an upper confidence bound with a higher or a lower level of confidence? Briefly explain why. Then compute a 99.95% upper confidence bound on the average monthly claim amount.

Answers

Answer:

a ) Upper bound of CI  is 7803483.87

b) The new upper bound is  8039767.74

Step-by-step explanation:

From sample data:

sample size    n  =  60

sample mean  x = 7500000

Sample standard deviation   s  =  1200000

a) Confidence Interval 95 %  then  significance level  α = 5 % o  α = 0.05

α/2 = 0.025     z(c) from z-table is  z(c) = 1.96

CI  95 % =  (   x  ±  z(c) * s/√n

CI  95 % =  (  7500000  ±  1.96 * 1200000/√60

CI  95 % =  (  7500000  ±  303483.87 )

CI  95 % =  ( 7196516.13  ;  7803483.87)

Then upper bound of CI  is 7803483.87

b) The company has to decrease the significance level equivalent to widen the confidence interval.

If CI now is   99.95 %    significance level is α = 0.0005 and

α/2 = 0.00025     z(c) for that α/2    is from z-table  z(c) ≈ 3.486

CI 99.95 %  = ( x ±  z(c)*s/√n

CI 99.95 %  =  7500000 ±  3.486*1200000/ √60

CI 99.95 %  =  (7500000 ±539767.74)

CI 99.95 %  = ( 6960232.26 ; 8039767.74)

The new upper bound is  8039767.74

which exponential equation is equivalent to the logarithmic equation below? log478=a
A. 478^10 = a
B. a^10 = 478
C. 478^a = 10
D. 10^a = 478

Answers

I think the answer is D

The base of a triangle is 9cm correct to the nearest cm.
The area of this triangle is 40 cm2 correct to the nearest 5 cm?.
Calculate the upper bound for the perpendicular height of this triangle.

Answers

Answer:

can you show the picture plz

Part
The number 3.14 is a rounded value of the mathematical quantity . If you replace 3.14 with 1, what is the exact formula for the circumference in
terms of the diameter, d, and in terms of the radius, r?
B IV x x
Font Sizes
А
E1 E2
Characters used: 0/15000
Submit
I need help

Answers

Answer:

C = 2(3.14)r

C = (3.14)d

Step-by-step explanation:

Circumference in terms of radius:

[tex]c \: = 2\pi \: r[/tex]

So replacing pi with 3.14 you would get 2(3.14)r

Circumference in terms of diameter:

[tex]c = d\pi[/tex]

So replacing pi with 3.14 you would get d(3.14)

Consider the following equation. -2x + 6 =[-2/3] + 5

Answers

Answer:

that's my answer

hope it helps

f(x)=2x^2-x-6 g(x)=4-x (f+g)(x)

Answers

[tex] \large \boxed{(f + g)(x) = f(x) + g(x)}[/tex]

The property above is distribution property where we distribute x-term in the function.

Substitute both f(x) and g(x) in.

[tex] \large{ \begin{cases} f(x) = 2 {x}^{2} - x - 6 \\ g(x) = 4 - x \end{cases}} \\ \large{f(x) + g(x) = (2 {x}^{2} - x - 6) + (4 - x)} \\ \large{f(x) + g(x) = 2 {x}^{2} - x - 6 + 4 - x}[/tex]

Évaluate/Combine like terms.

[tex] \large{f(x) + g(x) = 2 {x}^{2} - 2x - 2} [/tex]

The function can be factored so there are two answers. (Both of them work as one of them is factored form while the other one is not.)

[tex] \large{(f + g)(x) = 2 {x}^{2} -2x -2}[/tex]

Alternative

[tex] \large{(f + g)(x) = 2({x}^{2} -x - 1)}[/tex]

Answer

(f+g)(x) = 2x²-2x-2(f+g)(x) = 2(x²-x-1)

Both answers work. The second answer is in factored form.

Let me know if you have any doubts!

The first side of a triangle measures 4 in. less than the second side, the third side is 3 in more than the first side, and the perimeter is 15 in. How long is the third side?


If s represents the length of the second side, which of the following represents the length of the third side?
O S-4
OS-1
OS+3

Answers

Answer:

s-1

Step-by-step explanation:

1st side:  s-4

2nd side: s-4+3, which is 's-1'

ANSWER ASAP!! Use the substitution method to solve the system of equations. 3x+7y=1 and y=x-7

Answers

Answer:

x=5

y = -2

(5,-2)

Step-by-step explanation:

3x+7y=1 and y=x-7

Substitute the second equation in for y in the first equation

3x+7( x-7)=1

Distribute

3x+ 7x - 49 = 1

Combine like terms

10x - 49 =1

Add 49 to each side

10x-49+49 = 1+49

10x = 50

Divide by 10

10x/10 = 50/10

x = 5

Now find y

y = x-7

y = 5-7

y = -2

what is the largest six-digit base five number? How do you know?

Answers

Answer:

999999.9>repeating

Step-by-step explanation:

.1 more and it will be 7 digits and not 6 Hope this helps :D

9514 1404 393

Answer:

  444444₅ = 15624₁₀

Step-by-step explanation:

It will be 1 less than the smallest 7-digit base-5 number. That number is 5^6 = 15625, so the largest 6-digit base 5 number is 15624 in decimal. In base 5, it is 6 of the largest base-5 digits:

  444444₅ = 15624₁₀

__

The digits of the number cannot be any larger than the largest base-5 digit. The base-5 digits are 0 to 4, so the largest is 4.

PLS HELP ASAP!

Shown below is a cylinder. Calculate the volume using 3.14 for π

Answers

Answer:

V = 678.24 cm^3

Step-by-step explanation:

Formula for volume of a cylinder:

the radius is half of the diameter (12/2 = 6)

[tex]V = \pi r^{2} h\\V = \pi 6^{2} 6\\\\V = 678.24[/tex] cm^3

Answer:

678.24 cm^3

Step-by-step explanation:

The volume of a cylinder is given by

V  = pi r^2 h  where r is the radius and h is the height

We know the diameter

r =d/2 =  12/2 = 6

V = 3.14 * 6^2 * 6

678.24 cm^3

2x^2 + 3x - 12 when x = 5 help pls

Answers

2x^2 + 3(5)-12
2x^2+ 15+-12

combine like terms

2x^2+15+-12
(2x^2)+(15+-12)

Answer:
2x^2 + 3

[tex]\huge\textsf{Hey there!}[/tex]

[tex]\mathsf{2x^2 + 3x - 12}[/tex]

[tex]\mathsf{= 2(5)^2 + 3(5) - 12}[/tex]

[tex]\mathsf{5^2}[/tex]

[tex]\mathsf{= 5 \times 5}[/tex]

[tex]\mathsf{\bf = 25}[/tex]

[tex]\mathsf{2(25) + 3(5) - 12}[/tex]

[tex]\mathsf{2(25)}[/tex]

[tex]\mathsf{= \bf 50}[/tex]

[tex]\mathsf{3(5)}[/tex]

[tex]\mathsf{= \bf 15}[/tex]

[tex]\mathsf{= 50 + 15 - 12}[/tex]

[tex]\mathsf{50 + 15}[/tex]

[tex]\mathsf{= \bf 65}[/tex]

65 - 12

[tex]\mathsf{= \bf 53}[/tex]

[tex]\boxed{\boxed{\huge\textsf{Answer: \bf 53}}}\huge\checkmark[/tex]

[tex]\large\textsf{Good luck on your assignment and enjoy your day!}[/tex]

~[tex]\frak{Amphitrite1040:)}[/tex]

Answer and I will give you brainiliest ​

Answers

Answer:

x = 60 degree

Step-by-step explanation:

90 + 90 + 2x - 10 + 2x + 2x + 10 = 540 degree (sum of 5 interior angles)

180 + 6x = 540

6x = 540 - 180

x = 360/6

x = 60 degree

PLEASE HELP ME I WILL MARK TOU JT YOU HELP ME!!!

Answers

Given:

The figure of a construction.

To find:

The correct option that represents the given construction.

Solution:

In the given figure, the given angle is angle CAB.

The steps for the given figure are:

1. Draw a ray PQ.

2. Mark an arc BC on the given angle and mark the same arc on the line and the intersection of line and arc is point Q.

3. Set the compass of the length BA.

4. Put the compass on the point Q mark arc that intersect the first arc. The intersection of arcs is R.

5. Draw a ray PR.

These are the steps of construction to copy an angle.

Therefore, the correct option is A.

Matthew collected data on what types of pets sixth grade students owned.
He asked 200 students and then created this graph.
Based on the circle graph, how many students own a dog?

Answers

Can you please repost with the image

3.
June has 248 oranges. She sells of these oranges. How many oranges did June sell?
Answer.​

Answers

Answer:

93. Divide 248 by 8. One eighth of 248 = 31.

June sold 3/8 of the 248 oranges.

If 1/8 = 31 and June sold 3/8 of the oranges.

Multiply 3 × 31. Therefore June sold 93 oranges.

Step-by-step explanation:

Answer:

93 oranges

Step-by-step explanation:

3/8 of 248

[tex]\frac{y}{248} :\frac{\frac{3}{8} }{1}[/tex]

y × 1 = 248 × 3/8

y = 93

Use the listing method to write the following set \ 1,2,3\; [0, 1, 2, 3] O (1, 2)

Answers

0,1 is everything but you have to be 3/2 to know what your doing

If the measure of angle 2 Is (4x+10)degrees and the measure of angle 3 is (3x-5)degrees, what is the measure of angle 2 in degrees?

Answers

Answer:

110°

Step-by-step explanation:

4x+10 + (3x-5) = 180

7x + 5 = 180

7x = 175

x = 25

∠2 = 4(25) + 10 = 110

The measure of angle 2 in degrees, when both lines are perpendicular, is approximately 58.56 degrees.

To find the measure of angle 2 when the lines are perpendicular, we can set up an equation. Since the angles are formed by perpendicular lines, we know that the sum of angle 2 and angle 3 will be equal to 90 degrees. Mathematically, we can write this as:

(4x + 10) + (3x - 5) = 90

Now, let's solve the equation to find the value of x. First, combine like terms:

4x + 3x + 10 - 5 = 90

7x + 5 = 90

Next, isolate x by subtracting 5 from both sides:

7x = 85

Finally, divide by 7 to solve for x:

x = 85 / 7 ≈ 12.14

Now that we have the value of x, we can find the measure of angle 2 by substituting x back into its expression:

Angle 2 = 4x + 10

Angle 2 = 4(12.14) + 10

Angle 2 ≈ 48.56 + 10

Angle 2 ≈ 58.56 degrees

To know more about angle here

https://brainly.com/question/4316040

#SPJ2

A certain group of test subjects had pulse rates with a mean of 75.2 beats per minute and a standard deviation of 11.2 beats per minute. Use the range rule of thumb to identify the limits separating values that are significantly low or significantly high. Is a pulse rate of 147.6 beats per minute significantly low or significantly​ high?

Answers

Answer:

The z-score for a pulse rate of 147.6 beats per minute is 6.46 > 2, which means that this pulse rate is significantly high.

Step-by-step explanation:

Normal Probability Distribution:

Problems of normal distributions can be solved using the z-score formula.

In a set with mean [tex]\mu[/tex] and standard deviation [tex]\sigma[/tex], the z-score of a measure X is given by:

[tex]Z = \frac{X - \mu}{\sigma}[/tex]

The Z-score measures how many standard deviations the measure is from the mean. After finding the Z-score, we look at the z-score table and find the p-value associated with this z-score. This p-value is the probability that the value of the measure is smaller than X, that is, the percentile of X. Subtracting 1 by the p-value, we get the probability that the value of the measure is greater than X.

By the Range Rule of Thumb, if Z < -2, the measure X is significantly low, and if Z > 2, the measure X is significantly high.

Mean of 75.2 beats per minute and a standard deviation of 11.2 beats per minute.

This means that [tex]\mu = 75.2, \sigma = 11.2[/tex]

Is a pulse rate of 147.6 beats per minute significantly low or significantly​ high?

We have to find Z when X = 147.6. So

[tex]Z = \frac{X - \mu}{\sigma}[/tex]

[tex]Z = \frac{147.6 - 75.2}{11.2}[/tex]

[tex]Z = 6.46[/tex]

The z-score for a pulse rate of 147.6 beats per minute is 6.46 > 2, which means that this pulse rate is significantly high.

kelly bought two dozen apples she found 1/3 of them were rotten how manu apples were rotten

Answers

the answer is 8 apples were rotten

Answer:

8 apples are rotten.

Step-by-step explanation:

2*12=24 . There are 12 apples in a dozen.

24/3=8

Plz help me solve this algebra

Answers

Answer:

Step-by-step explanation:

[tex]16^{\frac{24}{64}} =16^{\frac{4*2*3}{4*2*8}}\\\\=(2^{4})^{\frac{3}{8}}\\\\=2^{4*\frac{3}{8}}\\\\=2^{\frac{3}{2}}=2^{3*\frac{1}{2}}\\\\=\sqrt{2*2*2}\\\\=2\sqrt{2}[/tex]

Which expression is equivalent to 1/2 (6x + 1/2)

Answers

Answer:

3x + 1/4

Step-by-step explanation:

1/2 (6x + 1/2)

Distribute

1/2 * 6x + 1/2 * 1/2

3x + 1/4

Write an augmented matrix of each system. Then, solve each system using matrix notation. Describe the solution set in vector notation.

Answers

Answer:

The answer is below

Step-by-step explanation:

Given the system of equations:

x + y + 2z = 9

2x + 4y - 3z = 1

3x + 6y - 5z = 0

This system of equation can be solved using matrix. This done by first representing the equations as matrix and then solving:

The matrix form is:

[tex]\left[\begin{array}{ccc}1&1&2\\2&4&-3\\3&6&-5\end{array}\right] \left[\begin{array}{c}x\\y\\z\end{array}\right] = \left[\begin{array}{c}9\\1\\0\end{array}\right] \\\\\\ \left[\begin{array}{c}x\\y\\z\end{array}\right] =\left[\begin{array}{ccc}1&1&2\\2&4&-3\\3&6&-5\end{array}\right] ^{-1} \left[\begin{array}{c}9\\1\\0\end{array}\right] \\\\\\[/tex]

[tex]\left[\begin{array}{c}x\\y\\z\end{array}\right] =\left[\begin{array}{ccc}2&-17&11\\-1&11&-7\\0&3&-2\end{array}\right] \left[\begin{array}{c}9\\1\\0\end{array}\right] \\\\\\ \left[\begin{array}{c}x\\y\\z\end{array}\right] =\left[\begin{array}{c}1\\2\\3\end{array}\right][/tex]

can someone help me solve this pls ? solve |x| + 7 < 4

Answers

Answer:

no solution

Step-by-step explanation:

|x| + 7 < 4

Subtract 7 from each side

|x| + 7-7 < 4-7

|x| < -3

But an absolute value must be greater than or equal to zero

There is no solution

Answer: No solutions

Let's solve the inequality step-by-step

[tex]|x|+7<4[/tex]

First subtract 7 from both sides

[tex]|x|+7-7<4-7\\|x|<-3[/tex]

Now usually you'd solve absolute value, but absolute value cannot be less than 0 so there can't be a solution.

Other Questions
The probability distribution of damage claims paid by Insurance ABC on collision insurance is as followed: Payment ($) Probability 0 0.85 500 0.04 1000 0.04 3000 0.03 5000 0.02 8000 0.01 10000 0.01 How much should the collision insurance premium be so the company can break even Hi. How does this drawing look like. I am an 11 year old kid by the way. Please tell the truth. I'll appreciate any advice. Please don't report. The endpoints of DEF are D(1, 4) and F(16, 14).Determine and state the coordinates of point E, ifDE: EF = 2:3. Ethan is installing a new tile backsplash in his kitchen. The tile he likes costs $3.50 per square foot. The area he is tiling is 36.5 square feet. How much will Ethan pay for the tile for his backsplash? Simplify the expression. 4^0 be careful i think this is a trick question 116,02 LC)Which of the triangles shown are obtuse triangles? (2 points)A. Triangle A and Triangle BB. Triangle B and Triangle DC. Triangle CD. Triangle A, Triangle B, and Triangle C Living organisms are commonly divided into six kingdoms: archaea, bacteria, fungi, protists,plants, and animals. Whats the biggest problem with classifying organisms in this way? Plz do 6 and 9 Plz Transfer of thermal energy between air molecules in closed room is an example ofconduction convection radiationAnswer and I will give you brainiliest Which statement best describes the narrators perspective in the excerpt? She is amused by the unusual fashions that factory girls once wore. She is proud of the mill-girls contribution to political events in her youth. She is impressed by the fame that the mill-girls achieved in her youth. She is sentimental about the friendships that have endured throughout her life. Explica qu son los codones y los anticodones La siguiente secuencia de nucletidos de ADN codifica para una secuencia de aminocidos que forman una protena hipottica, encuentre la secuencia de codones, anticodones y aminocidos que se forman 5 ` A T G A G C A C C C A A A C T T G C TC T T A T T C T A A A A A G A C T 3 Which of these statements is correct? The system of linear equations 6 x minus 5 y = 8 and 12 x minus 10 y = 16 has no solution. The system of linear equations 7 x + 2 y = 6 and 14 x + 4 y = 16 has an infinite number of solutions. The system of linear equations 8 x minus 3 y = 10 and 16 x minus 6 y = 22 has no solution. The system of linear equations 9 x + 6 y = 14 and 18 x + 12 y = 26 has an infinite number of solutions State your claim. Make sure your claim fully explains how infrared photography can be used to visualize temperature differences NOW NA PO PLS HELP !!!11.All of the following are part of consumer health except:health carehealth serviceshealth care facilitieshealth information2.Preventing and controlling the spread of diseases is one of the priority of health service TrueFalse3.What is Health Services?These are programs for government employeesThese are programs that aims to appraise the health conditions of individualsThese are programs used to train the athletesThese are programs that follows rules and regulations to adopt the New Normal When the Aryans migrated to India, but influential concept and they bring to the country a study is planned to compare the proportion of men who dislike anchovies with the proportion of women who dislike anchovies. the study seeks to determine if the proportions of men and women who dislike anchovies are different. a sample of 41 men was taken and the p^ estimate for the true proportion of men who dislike anchovies was determined to be 0.67. a sample of 56 women was also taken and the p^ estimate for the true proportion of women who dislike anchovies was determined to be 0.84. are the requirements satisfied to perform this hypothesis test What is the mean of the data? The amount of ice cream dispensed from a machine at an ice cream shop is normallydistributed. If the machine is used 800 times in a day, how many times did themachine dispense an amount that falls within three standard deviations from themean amount?A 798B 760C 544D 267 Frames control what displays on the Stage, while keyframes help to set up Due to people askingquestionsabout the cause of the black death, theirauthority had been ceased.A. King JohnB. The NoblesC. The Christian churchD. The Middle Class